Difference between revisions of "2021 WSMO Accuracy Round Problems/Problem 7"
(7 intermediate revisions by the same user not shown) | |||
Line 1: | Line 1: | ||
− | ==Problem | + | ==Problem== |
Find the value of <math>\sum_{n=1}^{100}\left(\sum_{i=1}^{n}r_i\right),</math> where <math>r_i</math> is the remainder when <math>2^i+3^i</math> is divided by 10. | Find the value of <math>\sum_{n=1}^{100}\left(\sum_{i=1}^{n}r_i\right),</math> where <math>r_i</math> is the remainder when <math>2^i+3^i</math> is divided by 10. | ||
==Solution 1== | ==Solution 1== | ||
From Fermat's Little Theorem, we find that <math>a^n\pmod{10}</math> is periodic in cycles of 4. This means that <math>a^n\equiv a^{n+4}\pmod{10}</math> for all <math>a,n.</math> Now, we will compute the first 4 values of <math>2^n+3^n\pmod{10}.</math> | From Fermat's Little Theorem, we find that <math>a^n\pmod{10}</math> is periodic in cycles of 4. This means that <math>a^n\equiv a^{n+4}\pmod{10}</math> for all <math>a,n.</math> Now, we will compute the first 4 values of <math>2^n+3^n\pmod{10}.</math> | ||
+ | <cmath> | ||
\begin{align*} | \begin{align*} | ||
2^1+3^1\equiv2+3\equiv&\text{ }5\pmod{10}\\ | 2^1+3^1\equiv2+3\equiv&\text{ }5\pmod{10}\\ | ||
Line 9: | Line 10: | ||
2^3+3^3\equiv8+27\equiv35\equiv&\text{ }5\pmod{10}\\ | 2^3+3^3\equiv8+27\equiv35\equiv&\text{ }5\pmod{10}\\ | ||
2^4+3^4\equiv16+81\equiv97\equiv&\text{ }7\pmod{10}\\ | 2^4+3^4\equiv16+81\equiv97\equiv&\text{ }7\pmod{10}\\ | ||
− | \end{align*} | + | \end{align*} </cmath> |
Thus, | Thus, | ||
+ | <cmath> | ||
\begin{align*} | \begin{align*} | ||
− | 2^n+3^n\equiv&\text{ }5\pmod{10}\tag{for | + | 2^n+3^n\equiv&\text{ }5\pmod{10}\tag{for n\equiv1,3\pmod{4}}\\ |
− | 2^n+3^n\equiv&\text{ }3\pmod{10}\tag{for | + | 2^n+3^n\equiv&\text{ }3\pmod{10}\tag{for n\equiv2\pmod{4}}\\ |
− | 2^n+3^n\equiv&\text{ }7\pmod{10}\tag{for | + | 2^n+3^n\equiv&\text{ }7\pmod{10}\tag{for n\equiv0\pmod{4}}\\ |
− | \end{align*} | + | \end{align*} </cmath> |
This means that | This means that | ||
+ | <cmath> | ||
\begin{align*} | \begin{align*} | ||
\sum_{n=1}^{100}\left(\sum_{i=1}^{n}r_i\right)&=\sum_{n=4a,1\leq a\leq25}(20a)+\sum_{n=4a+1,0\leq a\leq24}(20a+5)+\sum_{n=4a+2,0\leq a\leq24}(20a+8)+\sum_{n=4a+3,0\leq a\leq24}(20a+13)\\ | \sum_{n=1}^{100}\left(\sum_{i=1}^{n}r_i\right)&=\sum_{n=4a,1\leq a\leq25}(20a)+\sum_{n=4a+1,0\leq a\leq24}(20a+5)+\sum_{n=4a+2,0\leq a\leq24}(20a+8)+\sum_{n=4a+3,0\leq a\leq24}(20a+13)\\ | ||
Line 24: | Line 27: | ||
&=6500+18000+650\\ | &=6500+18000+650\\ | ||
&=\boxed{25150}\\ | &=\boxed{25150}\\ | ||
− | \end{align*} | + | \end{align*}</cmath> |
~pinkpig | ~pinkpig | ||
Latest revision as of 10:24, 11 July 2022
Problem
Find the value of where is the remainder when is divided by 10.
Solution 1
From Fermat's Little Theorem, we find that is periodic in cycles of 4. This means that for all Now, we will compute the first 4 values of Thus,
\begin{align*} 2^n+3^n\equiv&\text{ }5\pmod{10}\tag{for n\equiv1,3\pmod{4}}\\ 2^n+3^n\equiv&\text{ }3\pmod{10}\tag{for n\equiv2\pmod{4}}\\ 2^n+3^n\equiv&\text{ }7\pmod{10}\tag{for n\equiv0\pmod{4}}\\ \end{align*} (Error compiling LaTeX. Unknown error_msg)
This means that ~pinkpig
Solution 2
We will begin by examining and :
and
From this, we can note that:
We can simplify our sum as follows:
Note that :
~BigKahuna227